IMO Shortlist 1998 problem A1


Kvaliteta:
  Avg: 4,3
Težina:
  Avg: 6,0
Dodao/la: arhiva
2. travnja 2012.
LaTeX PDF
Let a_{1},a_{2},\ldots ,a_{n} be positive real numbers such that a_{1}+a_{2}+\cdots +a_{n}<1. Prove that

\frac{a_{1} a_{2} \cdots a_{n} \left[ 1 - (a_{1} + a_{2} + \cdots + a_{n}) \right] }{(a_{1} + a_{2} + \cdots + a_{n})( 1 - a_1)(1 - a_2) \cdots (1 - a_n)} \leqslant \frac{1}{n^{n+1}}
Izvor: Međunarodna matematička olimpijada, shortlist 1998



Komentari:

ja sam rucno stavljao tag invalid gdje sam vidio da treba. nisam isao sve provjeravati.
sredeno.. kico, kako to da zadatak nije imao tag invalid?
Daj nek neko pogleda ovaj zadatak xd